Introduction to Quantum Mechanics 2nd Edition

Published by Cambridge University Press
ISBN 10: 1107179866
ISBN 13: 978-1-10717-986-8

Chapter 1 - Section 1.5 - Momentum - Problems - Page 18: 1.6

Answer

From Eq.(1.29), \begin{eqnarray} \label{1} \frac{d\langle x\rangle}{dt}=\int x \frac{\partial}{\partial t} \mid \Psi \mid ^{2} dx \end{eqnarray} In Eq.\eqref{1}, the L.H.S. shows the variation of $\langle x \rangle$ with respect to time because of the time dependence of probability density function $\mid \Psi\left(x,t\right)\mid^{2}$. From the chain rule, R.H.S. of Eq.\eqref{1} can be written in following manner: \begin{eqnarray} \label{2} \frac{d\langle x\rangle}{dt}&=&\int x \left( \frac{\partial \mid \Psi \mid ^{2}}{\partial x}\frac{\partial x}{\partial t}\right) dx \nonumber \\ \end{eqnarray} From integration by parts, \begin{eqnarray} \label{3} \frac{d\langle x\rangle}{dt}&=& x\frac{\partial x}{\partial t}\int \frac{\partial \mid \Psi \mid ^{2}}{\partial x} dx-\int \frac{d}{dx}\left(x\frac{\partial x}{\partial t}\right) \mid \Psi \mid ^{2} dx, \end{eqnarray} In Eq.\eqref{3}, the term $\frac{\partial x}{\partial t}$ vanishes as $x$ is independent of time '$t$'. Therefore, it implies $\frac{d\langle x\rangle}{dt}=0$. So it would not be appropriate to apply integration by parts for the solution of Eq.\eqref{1}.

Work Step by Step

From Eq.(1.29), \begin{eqnarray} \label{1} \frac{d\langle x\rangle}{dt}=\int x \frac{\partial}{\partial t} \mid \Psi \mid ^{2} dx \end{eqnarray} In Eq.\eqref{1}, the L.H.S. shows the variation of $\langle x \rangle$ with respect to time because of the time dependence of probability density function $\mid \Psi\left(x,t\right)\mid^{2}$. From the chain rule, R.H.S. of Eq.\eqref{1} can be written in following manner: \begin{eqnarray} \label{2} \frac{d\langle x\rangle}{dt}&=&\int x \left( \frac{\partial \mid \Psi \mid ^{2}}{\partial x}\frac{\partial x}{\partial t}\right) dx \nonumber \\ \end{eqnarray} From integration by parts, \begin{eqnarray} \label{3} \frac{d\langle x\rangle}{dt}&=& x\frac{\partial x}{\partial t}\int \frac{\partial \mid \Psi \mid ^{2}}{\partial x} dx-\int \frac{d}{dx}\left(x\frac{\partial x}{\partial t}\right) \mid \Psi \mid ^{2} dx, \end{eqnarray} In Eq.\eqref{3}, the term $\frac{\partial x}{\partial t}$ vanishes as $x$ is independent of time '$t$'. Therefore, it implies $\frac{d\langle x\rangle}{dt}=0$. So it would not be appropriate to apply integration by parts for the solution of Eq.\eqref{1}.
Update this answer!

You can help us out by revising, improving and updating this answer.

Update this answer

After you claim an answer you’ll have 24 hours to send in a draft. An editor will review the submission and either publish your submission or provide feedback.